How should I solve this integral with changing parameters?












4












$begingroup$


I can't solve this. How should I proceed?




$$iint_De^{largefrac{y-x}{y+x}}mathrm dxmathrm dy$$




$D$ is the triangle with these coordinates $(0,0), (0,2), (2,0)$ and I've changed the parameters this way $u=y-x$ and $v= y+x$ and the Jacobian is $-1/2$ but I have problem finding the range of $u$ and $v$ to calculate the integral










share|cite|improve this question









New contributor




khoshrang is a new contributor to this site. Take care in asking for clarification, commenting, and answering.
Check out our Code of Conduct.







$endgroup$








  • 1




    $begingroup$
    Duplicate (with an enlarged domain) of math.stackexchange.com/q/845996 and math.stackexchange.com/q/408558
    $endgroup$
    – Jean Marie
    5 hours ago










  • $begingroup$
    i couldn't find them at first but even after your mention i couldn't find the elaboration to draw the new region to find the new range for parameters but thanks !
    $endgroup$
    – khoshrang
    4 hours ago
















4












$begingroup$


I can't solve this. How should I proceed?




$$iint_De^{largefrac{y-x}{y+x}}mathrm dxmathrm dy$$




$D$ is the triangle with these coordinates $(0,0), (0,2), (2,0)$ and I've changed the parameters this way $u=y-x$ and $v= y+x$ and the Jacobian is $-1/2$ but I have problem finding the range of $u$ and $v$ to calculate the integral










share|cite|improve this question









New contributor




khoshrang is a new contributor to this site. Take care in asking for clarification, commenting, and answering.
Check out our Code of Conduct.







$endgroup$








  • 1




    $begingroup$
    Duplicate (with an enlarged domain) of math.stackexchange.com/q/845996 and math.stackexchange.com/q/408558
    $endgroup$
    – Jean Marie
    5 hours ago










  • $begingroup$
    i couldn't find them at first but even after your mention i couldn't find the elaboration to draw the new region to find the new range for parameters but thanks !
    $endgroup$
    – khoshrang
    4 hours ago














4












4








4





$begingroup$


I can't solve this. How should I proceed?




$$iint_De^{largefrac{y-x}{y+x}}mathrm dxmathrm dy$$




$D$ is the triangle with these coordinates $(0,0), (0,2), (2,0)$ and I've changed the parameters this way $u=y-x$ and $v= y+x$ and the Jacobian is $-1/2$ but I have problem finding the range of $u$ and $v$ to calculate the integral










share|cite|improve this question









New contributor




khoshrang is a new contributor to this site. Take care in asking for clarification, commenting, and answering.
Check out our Code of Conduct.







$endgroup$




I can't solve this. How should I proceed?




$$iint_De^{largefrac{y-x}{y+x}}mathrm dxmathrm dy$$




$D$ is the triangle with these coordinates $(0,0), (0,2), (2,0)$ and I've changed the parameters this way $u=y-x$ and $v= y+x$ and the Jacobian is $-1/2$ but I have problem finding the range of $u$ and $v$ to calculate the integral







definite-integrals






share|cite|improve this question









New contributor




khoshrang is a new contributor to this site. Take care in asking for clarification, commenting, and answering.
Check out our Code of Conduct.











share|cite|improve this question









New contributor




khoshrang is a new contributor to this site. Take care in asking for clarification, commenting, and answering.
Check out our Code of Conduct.









share|cite|improve this question




share|cite|improve this question








edited 7 hours ago









mrtaurho

5,74551540




5,74551540






New contributor




khoshrang is a new contributor to this site. Take care in asking for clarification, commenting, and answering.
Check out our Code of Conduct.









asked 7 hours ago









khoshrangkhoshrang

253




253




New contributor




khoshrang is a new contributor to this site. Take care in asking for clarification, commenting, and answering.
Check out our Code of Conduct.





New contributor





khoshrang is a new contributor to this site. Take care in asking for clarification, commenting, and answering.
Check out our Code of Conduct.






khoshrang is a new contributor to this site. Take care in asking for clarification, commenting, and answering.
Check out our Code of Conduct.








  • 1




    $begingroup$
    Duplicate (with an enlarged domain) of math.stackexchange.com/q/845996 and math.stackexchange.com/q/408558
    $endgroup$
    – Jean Marie
    5 hours ago










  • $begingroup$
    i couldn't find them at first but even after your mention i couldn't find the elaboration to draw the new region to find the new range for parameters but thanks !
    $endgroup$
    – khoshrang
    4 hours ago














  • 1




    $begingroup$
    Duplicate (with an enlarged domain) of math.stackexchange.com/q/845996 and math.stackexchange.com/q/408558
    $endgroup$
    – Jean Marie
    5 hours ago










  • $begingroup$
    i couldn't find them at first but even after your mention i couldn't find the elaboration to draw the new region to find the new range for parameters but thanks !
    $endgroup$
    – khoshrang
    4 hours ago








1




1




$begingroup$
Duplicate (with an enlarged domain) of math.stackexchange.com/q/845996 and math.stackexchange.com/q/408558
$endgroup$
– Jean Marie
5 hours ago




$begingroup$
Duplicate (with an enlarged domain) of math.stackexchange.com/q/845996 and math.stackexchange.com/q/408558
$endgroup$
– Jean Marie
5 hours ago












$begingroup$
i couldn't find them at first but even after your mention i couldn't find the elaboration to draw the new region to find the new range for parameters but thanks !
$endgroup$
– khoshrang
4 hours ago




$begingroup$
i couldn't find them at first but even after your mention i couldn't find the elaboration to draw the new region to find the new range for parameters but thanks !
$endgroup$
– khoshrang
4 hours ago










3 Answers
3






active

oldest

votes


















7












$begingroup$

Start by making a drawing of your domain. You can see that the $v$ is along the diagonal in the first quadrant, and $u$ is along the diagonal in the second quadrant. You can also see that the line between $(0,2)$ and $(2,0)$ is parallel to $u$, and intersects $v$ axis at $v=1$. So $v$ varies between $0$ and $1$ and $u$ varies between $-v$ and $v$.






share|cite|improve this answer









$endgroup$













  • $begingroup$
    thank you for your answer but my problem is that i can't draw my domain how should i draw u and v vectors along the y and x?
    $endgroup$
    – khoshrang
    5 hours ago






  • 1




    $begingroup$
    You draw a triangle $(0,0), (2,0),(0,2)$. The $x$ axis is from $(0,0)$ to $(2,0)$. What are $u$ and $v$? You have $u=y-x$ and $v=y+x$. So if I want to draw the $u$ axis you need to go from $(0,0)$ to $u=1$ and $v=0$. The second equation means $v=0=y+x$ or $x=-y$. If you plug into $u=1=y-x=y-(-y)$ you get $y=1/2$ and then $x=-1/2$. That means that you go in the second quadrant, along the $y=-x$. Similarly, the $v$ axis means $v=1$ and $u=0$. $u=0$ means $y=x$, which is the diagonal in the first quadrant.
    $endgroup$
    – Andrei
    5 hours ago










  • $begingroup$
    "So if I want to draw the u axis you need to go from (0,0) to u=1 and v=0 " why to u=1 and v=0?
    $endgroup$
    – khoshrang
    5 hours ago






  • 1




    $begingroup$
    If you want to draw the $x$ axis, you go from $(0,0)$ to $x=1$ and $y=0$. If you want to draw the $y$ axis, you go to $y=0$ and $x=0$.
    $endgroup$
    – Andrei
    5 hours ago






  • 1




    $begingroup$
    thank you i just got the whole idea and i'm gonna choose your answer because it directly answers the parameter change that i had done in the question
    $endgroup$
    – khoshrang
    4 hours ago



















6












$begingroup$

Your domain is $$D={(x,y):0<x<2,,,0<y<2,,,x+y<2}$$



Using change of variables $(x,y)to(u,v)$ with $$u=frac{x-y}{x+y},,,v=x+y$$



The region is now $$R={(u,v):-1<u<1,,0<v<2}$$



Therefore,



begin{align}
iint_D expleft({-frac{x-y}{x+y}}right),mathrm{d}x,mathrm{d}y&=frac{1}{2}iint_R ve^{-u},mathrm{d}u,mathrm{d}v
\\&=frac{1}{2}int_0^2v,mathrm{d}vint_{-1}^1 e^{-u},mathrm{d}u
end{align}






share|cite|improve this answer











$endgroup$













  • $begingroup$
    Finally the $v$ appears, but with no explanation at all?
    $endgroup$
    – NickD
    6 hours ago










  • $begingroup$
    @NickD I have left the calculation of the jacobian to the OP.
    $endgroup$
    – StubbornAtom
    6 hours ago










  • $begingroup$
    That's fine, but you should at least mention it, if only for the sake of future readers.
    $endgroup$
    – NickD
    6 hours ago










  • $begingroup$
    It is pretty clear as it is, I think.
    $endgroup$
    – StubbornAtom
    6 hours ago






  • 1




    $begingroup$
    thank you! your new parameters are better than i had chosen in question but i'm gonna choose the other answer because it directly answers to the parameters that i had mentioned in the question
    $endgroup$
    – khoshrang
    4 hours ago





















0












$begingroup$

Computer algebra gives (for the general case):



$$e left(x^2 text{Ei}left(-frac{2 x}{x+y}right)-frac{y^2 text{Ei}left(frac{2
y}{x+y}right)}{e^2}right)+frac{1}{2} e^{1-frac{2 x}{x+y}} (x+y)^2$$



over your specified region:



$$e-frac{1}{e}$$






share|cite|improve this answer









$endgroup$













  • $begingroup$
    yes the final answer is right due to my answer sheet but i should solve this by hand not computer any idea of how should i solve this on paper?
    $endgroup$
    – khoshrang
    7 hours ago












  • $begingroup$
    (-1) because this is not an answer to the question. See the other excellent responses!
    $endgroup$
    – Jimmy Sabater
    6 hours ago










  • $begingroup$
    @JimmySabater: I find it absolutely astounding that the "solution" (+7 and accept) lists the trivial point of how to draw a triangle, while the full correct solution leads you to downvote.
    $endgroup$
    – David G. Stork
    4 hours ago










  • $begingroup$
    The accepted solution takes the time to actually reply the steps to the asker. Your answer is not one bit helpful to be honest and when asker asked you a question you didnt even reply... that si the reason the accepted answer got many upvotes and the other answer too which also boh actually answer tbe qestion
    $endgroup$
    – Neymar
    4 hours ago











Your Answer





StackExchange.ifUsing("editor", function () {
return StackExchange.using("mathjaxEditing", function () {
StackExchange.MarkdownEditor.creationCallbacks.add(function (editor, postfix) {
StackExchange.mathjaxEditing.prepareWmdForMathJax(editor, postfix, [["$", "$"], ["\\(","\\)"]]);
});
});
}, "mathjax-editing");

StackExchange.ready(function() {
var channelOptions = {
tags: "".split(" "),
id: "69"
};
initTagRenderer("".split(" "), "".split(" "), channelOptions);

StackExchange.using("externalEditor", function() {
// Have to fire editor after snippets, if snippets enabled
if (StackExchange.settings.snippets.snippetsEnabled) {
StackExchange.using("snippets", function() {
createEditor();
});
}
else {
createEditor();
}
});

function createEditor() {
StackExchange.prepareEditor({
heartbeatType: 'answer',
autoActivateHeartbeat: false,
convertImagesToLinks: true,
noModals: true,
showLowRepImageUploadWarning: true,
reputationToPostImages: 10,
bindNavPrevention: true,
postfix: "",
imageUploader: {
brandingHtml: "Powered by u003ca class="icon-imgur-white" href="https://imgur.com/"u003eu003c/au003e",
contentPolicyHtml: "User contributions licensed under u003ca href="https://creativecommons.org/licenses/by-sa/3.0/"u003ecc by-sa 3.0 with attribution requiredu003c/au003e u003ca href="https://stackoverflow.com/legal/content-policy"u003e(content policy)u003c/au003e",
allowUrls: true
},
noCode: true, onDemand: true,
discardSelector: ".discard-answer"
,immediatelyShowMarkdownHelp:true
});


}
});






khoshrang is a new contributor. Be nice, and check out our Code of Conduct.










draft saved

draft discarded


















StackExchange.ready(
function () {
StackExchange.openid.initPostLogin('.new-post-login', 'https%3a%2f%2fmath.stackexchange.com%2fquestions%2f3141532%2fhow-should-i-solve-this-integral-with-changing-parameters%23new-answer', 'question_page');
}
);

Post as a guest















Required, but never shown

























3 Answers
3






active

oldest

votes








3 Answers
3






active

oldest

votes









active

oldest

votes






active

oldest

votes









7












$begingroup$

Start by making a drawing of your domain. You can see that the $v$ is along the diagonal in the first quadrant, and $u$ is along the diagonal in the second quadrant. You can also see that the line between $(0,2)$ and $(2,0)$ is parallel to $u$, and intersects $v$ axis at $v=1$. So $v$ varies between $0$ and $1$ and $u$ varies between $-v$ and $v$.






share|cite|improve this answer









$endgroup$













  • $begingroup$
    thank you for your answer but my problem is that i can't draw my domain how should i draw u and v vectors along the y and x?
    $endgroup$
    – khoshrang
    5 hours ago






  • 1




    $begingroup$
    You draw a triangle $(0,0), (2,0),(0,2)$. The $x$ axis is from $(0,0)$ to $(2,0)$. What are $u$ and $v$? You have $u=y-x$ and $v=y+x$. So if I want to draw the $u$ axis you need to go from $(0,0)$ to $u=1$ and $v=0$. The second equation means $v=0=y+x$ or $x=-y$. If you plug into $u=1=y-x=y-(-y)$ you get $y=1/2$ and then $x=-1/2$. That means that you go in the second quadrant, along the $y=-x$. Similarly, the $v$ axis means $v=1$ and $u=0$. $u=0$ means $y=x$, which is the diagonal in the first quadrant.
    $endgroup$
    – Andrei
    5 hours ago










  • $begingroup$
    "So if I want to draw the u axis you need to go from (0,0) to u=1 and v=0 " why to u=1 and v=0?
    $endgroup$
    – khoshrang
    5 hours ago






  • 1




    $begingroup$
    If you want to draw the $x$ axis, you go from $(0,0)$ to $x=1$ and $y=0$. If you want to draw the $y$ axis, you go to $y=0$ and $x=0$.
    $endgroup$
    – Andrei
    5 hours ago






  • 1




    $begingroup$
    thank you i just got the whole idea and i'm gonna choose your answer because it directly answers the parameter change that i had done in the question
    $endgroup$
    – khoshrang
    4 hours ago
















7












$begingroup$

Start by making a drawing of your domain. You can see that the $v$ is along the diagonal in the first quadrant, and $u$ is along the diagonal in the second quadrant. You can also see that the line between $(0,2)$ and $(2,0)$ is parallel to $u$, and intersects $v$ axis at $v=1$. So $v$ varies between $0$ and $1$ and $u$ varies between $-v$ and $v$.






share|cite|improve this answer









$endgroup$













  • $begingroup$
    thank you for your answer but my problem is that i can't draw my domain how should i draw u and v vectors along the y and x?
    $endgroup$
    – khoshrang
    5 hours ago






  • 1




    $begingroup$
    You draw a triangle $(0,0), (2,0),(0,2)$. The $x$ axis is from $(0,0)$ to $(2,0)$. What are $u$ and $v$? You have $u=y-x$ and $v=y+x$. So if I want to draw the $u$ axis you need to go from $(0,0)$ to $u=1$ and $v=0$. The second equation means $v=0=y+x$ or $x=-y$. If you plug into $u=1=y-x=y-(-y)$ you get $y=1/2$ and then $x=-1/2$. That means that you go in the second quadrant, along the $y=-x$. Similarly, the $v$ axis means $v=1$ and $u=0$. $u=0$ means $y=x$, which is the diagonal in the first quadrant.
    $endgroup$
    – Andrei
    5 hours ago










  • $begingroup$
    "So if I want to draw the u axis you need to go from (0,0) to u=1 and v=0 " why to u=1 and v=0?
    $endgroup$
    – khoshrang
    5 hours ago






  • 1




    $begingroup$
    If you want to draw the $x$ axis, you go from $(0,0)$ to $x=1$ and $y=0$. If you want to draw the $y$ axis, you go to $y=0$ and $x=0$.
    $endgroup$
    – Andrei
    5 hours ago






  • 1




    $begingroup$
    thank you i just got the whole idea and i'm gonna choose your answer because it directly answers the parameter change that i had done in the question
    $endgroup$
    – khoshrang
    4 hours ago














7












7








7





$begingroup$

Start by making a drawing of your domain. You can see that the $v$ is along the diagonal in the first quadrant, and $u$ is along the diagonal in the second quadrant. You can also see that the line between $(0,2)$ and $(2,0)$ is parallel to $u$, and intersects $v$ axis at $v=1$. So $v$ varies between $0$ and $1$ and $u$ varies between $-v$ and $v$.






share|cite|improve this answer









$endgroup$



Start by making a drawing of your domain. You can see that the $v$ is along the diagonal in the first quadrant, and $u$ is along the diagonal in the second quadrant. You can also see that the line between $(0,2)$ and $(2,0)$ is parallel to $u$, and intersects $v$ axis at $v=1$. So $v$ varies between $0$ and $1$ and $u$ varies between $-v$ and $v$.







share|cite|improve this answer












share|cite|improve this answer



share|cite|improve this answer










answered 6 hours ago









AndreiAndrei

13.1k21230




13.1k21230












  • $begingroup$
    thank you for your answer but my problem is that i can't draw my domain how should i draw u and v vectors along the y and x?
    $endgroup$
    – khoshrang
    5 hours ago






  • 1




    $begingroup$
    You draw a triangle $(0,0), (2,0),(0,2)$. The $x$ axis is from $(0,0)$ to $(2,0)$. What are $u$ and $v$? You have $u=y-x$ and $v=y+x$. So if I want to draw the $u$ axis you need to go from $(0,0)$ to $u=1$ and $v=0$. The second equation means $v=0=y+x$ or $x=-y$. If you plug into $u=1=y-x=y-(-y)$ you get $y=1/2$ and then $x=-1/2$. That means that you go in the second quadrant, along the $y=-x$. Similarly, the $v$ axis means $v=1$ and $u=0$. $u=0$ means $y=x$, which is the diagonal in the first quadrant.
    $endgroup$
    – Andrei
    5 hours ago










  • $begingroup$
    "So if I want to draw the u axis you need to go from (0,0) to u=1 and v=0 " why to u=1 and v=0?
    $endgroup$
    – khoshrang
    5 hours ago






  • 1




    $begingroup$
    If you want to draw the $x$ axis, you go from $(0,0)$ to $x=1$ and $y=0$. If you want to draw the $y$ axis, you go to $y=0$ and $x=0$.
    $endgroup$
    – Andrei
    5 hours ago






  • 1




    $begingroup$
    thank you i just got the whole idea and i'm gonna choose your answer because it directly answers the parameter change that i had done in the question
    $endgroup$
    – khoshrang
    4 hours ago


















  • $begingroup$
    thank you for your answer but my problem is that i can't draw my domain how should i draw u and v vectors along the y and x?
    $endgroup$
    – khoshrang
    5 hours ago






  • 1




    $begingroup$
    You draw a triangle $(0,0), (2,0),(0,2)$. The $x$ axis is from $(0,0)$ to $(2,0)$. What are $u$ and $v$? You have $u=y-x$ and $v=y+x$. So if I want to draw the $u$ axis you need to go from $(0,0)$ to $u=1$ and $v=0$. The second equation means $v=0=y+x$ or $x=-y$. If you plug into $u=1=y-x=y-(-y)$ you get $y=1/2$ and then $x=-1/2$. That means that you go in the second quadrant, along the $y=-x$. Similarly, the $v$ axis means $v=1$ and $u=0$. $u=0$ means $y=x$, which is the diagonal in the first quadrant.
    $endgroup$
    – Andrei
    5 hours ago










  • $begingroup$
    "So if I want to draw the u axis you need to go from (0,0) to u=1 and v=0 " why to u=1 and v=0?
    $endgroup$
    – khoshrang
    5 hours ago






  • 1




    $begingroup$
    If you want to draw the $x$ axis, you go from $(0,0)$ to $x=1$ and $y=0$. If you want to draw the $y$ axis, you go to $y=0$ and $x=0$.
    $endgroup$
    – Andrei
    5 hours ago






  • 1




    $begingroup$
    thank you i just got the whole idea and i'm gonna choose your answer because it directly answers the parameter change that i had done in the question
    $endgroup$
    – khoshrang
    4 hours ago
















$begingroup$
thank you for your answer but my problem is that i can't draw my domain how should i draw u and v vectors along the y and x?
$endgroup$
– khoshrang
5 hours ago




$begingroup$
thank you for your answer but my problem is that i can't draw my domain how should i draw u and v vectors along the y and x?
$endgroup$
– khoshrang
5 hours ago




1




1




$begingroup$
You draw a triangle $(0,0), (2,0),(0,2)$. The $x$ axis is from $(0,0)$ to $(2,0)$. What are $u$ and $v$? You have $u=y-x$ and $v=y+x$. So if I want to draw the $u$ axis you need to go from $(0,0)$ to $u=1$ and $v=0$. The second equation means $v=0=y+x$ or $x=-y$. If you plug into $u=1=y-x=y-(-y)$ you get $y=1/2$ and then $x=-1/2$. That means that you go in the second quadrant, along the $y=-x$. Similarly, the $v$ axis means $v=1$ and $u=0$. $u=0$ means $y=x$, which is the diagonal in the first quadrant.
$endgroup$
– Andrei
5 hours ago




$begingroup$
You draw a triangle $(0,0), (2,0),(0,2)$. The $x$ axis is from $(0,0)$ to $(2,0)$. What are $u$ and $v$? You have $u=y-x$ and $v=y+x$. So if I want to draw the $u$ axis you need to go from $(0,0)$ to $u=1$ and $v=0$. The second equation means $v=0=y+x$ or $x=-y$. If you plug into $u=1=y-x=y-(-y)$ you get $y=1/2$ and then $x=-1/2$. That means that you go in the second quadrant, along the $y=-x$. Similarly, the $v$ axis means $v=1$ and $u=0$. $u=0$ means $y=x$, which is the diagonal in the first quadrant.
$endgroup$
– Andrei
5 hours ago












$begingroup$
"So if I want to draw the u axis you need to go from (0,0) to u=1 and v=0 " why to u=1 and v=0?
$endgroup$
– khoshrang
5 hours ago




$begingroup$
"So if I want to draw the u axis you need to go from (0,0) to u=1 and v=0 " why to u=1 and v=0?
$endgroup$
– khoshrang
5 hours ago




1




1




$begingroup$
If you want to draw the $x$ axis, you go from $(0,0)$ to $x=1$ and $y=0$. If you want to draw the $y$ axis, you go to $y=0$ and $x=0$.
$endgroup$
– Andrei
5 hours ago




$begingroup$
If you want to draw the $x$ axis, you go from $(0,0)$ to $x=1$ and $y=0$. If you want to draw the $y$ axis, you go to $y=0$ and $x=0$.
$endgroup$
– Andrei
5 hours ago




1




1




$begingroup$
thank you i just got the whole idea and i'm gonna choose your answer because it directly answers the parameter change that i had done in the question
$endgroup$
– khoshrang
4 hours ago




$begingroup$
thank you i just got the whole idea and i'm gonna choose your answer because it directly answers the parameter change that i had done in the question
$endgroup$
– khoshrang
4 hours ago











6












$begingroup$

Your domain is $$D={(x,y):0<x<2,,,0<y<2,,,x+y<2}$$



Using change of variables $(x,y)to(u,v)$ with $$u=frac{x-y}{x+y},,,v=x+y$$



The region is now $$R={(u,v):-1<u<1,,0<v<2}$$



Therefore,



begin{align}
iint_D expleft({-frac{x-y}{x+y}}right),mathrm{d}x,mathrm{d}y&=frac{1}{2}iint_R ve^{-u},mathrm{d}u,mathrm{d}v
\\&=frac{1}{2}int_0^2v,mathrm{d}vint_{-1}^1 e^{-u},mathrm{d}u
end{align}






share|cite|improve this answer











$endgroup$













  • $begingroup$
    Finally the $v$ appears, but with no explanation at all?
    $endgroup$
    – NickD
    6 hours ago










  • $begingroup$
    @NickD I have left the calculation of the jacobian to the OP.
    $endgroup$
    – StubbornAtom
    6 hours ago










  • $begingroup$
    That's fine, but you should at least mention it, if only for the sake of future readers.
    $endgroup$
    – NickD
    6 hours ago










  • $begingroup$
    It is pretty clear as it is, I think.
    $endgroup$
    – StubbornAtom
    6 hours ago






  • 1




    $begingroup$
    thank you! your new parameters are better than i had chosen in question but i'm gonna choose the other answer because it directly answers to the parameters that i had mentioned in the question
    $endgroup$
    – khoshrang
    4 hours ago


















6












$begingroup$

Your domain is $$D={(x,y):0<x<2,,,0<y<2,,,x+y<2}$$



Using change of variables $(x,y)to(u,v)$ with $$u=frac{x-y}{x+y},,,v=x+y$$



The region is now $$R={(u,v):-1<u<1,,0<v<2}$$



Therefore,



begin{align}
iint_D expleft({-frac{x-y}{x+y}}right),mathrm{d}x,mathrm{d}y&=frac{1}{2}iint_R ve^{-u},mathrm{d}u,mathrm{d}v
\\&=frac{1}{2}int_0^2v,mathrm{d}vint_{-1}^1 e^{-u},mathrm{d}u
end{align}






share|cite|improve this answer











$endgroup$













  • $begingroup$
    Finally the $v$ appears, but with no explanation at all?
    $endgroup$
    – NickD
    6 hours ago










  • $begingroup$
    @NickD I have left the calculation of the jacobian to the OP.
    $endgroup$
    – StubbornAtom
    6 hours ago










  • $begingroup$
    That's fine, but you should at least mention it, if only for the sake of future readers.
    $endgroup$
    – NickD
    6 hours ago










  • $begingroup$
    It is pretty clear as it is, I think.
    $endgroup$
    – StubbornAtom
    6 hours ago






  • 1




    $begingroup$
    thank you! your new parameters are better than i had chosen in question but i'm gonna choose the other answer because it directly answers to the parameters that i had mentioned in the question
    $endgroup$
    – khoshrang
    4 hours ago
















6












6








6





$begingroup$

Your domain is $$D={(x,y):0<x<2,,,0<y<2,,,x+y<2}$$



Using change of variables $(x,y)to(u,v)$ with $$u=frac{x-y}{x+y},,,v=x+y$$



The region is now $$R={(u,v):-1<u<1,,0<v<2}$$



Therefore,



begin{align}
iint_D expleft({-frac{x-y}{x+y}}right),mathrm{d}x,mathrm{d}y&=frac{1}{2}iint_R ve^{-u},mathrm{d}u,mathrm{d}v
\\&=frac{1}{2}int_0^2v,mathrm{d}vint_{-1}^1 e^{-u},mathrm{d}u
end{align}






share|cite|improve this answer











$endgroup$



Your domain is $$D={(x,y):0<x<2,,,0<y<2,,,x+y<2}$$



Using change of variables $(x,y)to(u,v)$ with $$u=frac{x-y}{x+y},,,v=x+y$$



The region is now $$R={(u,v):-1<u<1,,0<v<2}$$



Therefore,



begin{align}
iint_D expleft({-frac{x-y}{x+y}}right),mathrm{d}x,mathrm{d}y&=frac{1}{2}iint_R ve^{-u},mathrm{d}u,mathrm{d}v
\\&=frac{1}{2}int_0^2v,mathrm{d}vint_{-1}^1 e^{-u},mathrm{d}u
end{align}







share|cite|improve this answer














share|cite|improve this answer



share|cite|improve this answer








edited 6 hours ago

























answered 6 hours ago









StubbornAtomStubbornAtom

6,14311339




6,14311339












  • $begingroup$
    Finally the $v$ appears, but with no explanation at all?
    $endgroup$
    – NickD
    6 hours ago










  • $begingroup$
    @NickD I have left the calculation of the jacobian to the OP.
    $endgroup$
    – StubbornAtom
    6 hours ago










  • $begingroup$
    That's fine, but you should at least mention it, if only for the sake of future readers.
    $endgroup$
    – NickD
    6 hours ago










  • $begingroup$
    It is pretty clear as it is, I think.
    $endgroup$
    – StubbornAtom
    6 hours ago






  • 1




    $begingroup$
    thank you! your new parameters are better than i had chosen in question but i'm gonna choose the other answer because it directly answers to the parameters that i had mentioned in the question
    $endgroup$
    – khoshrang
    4 hours ago




















  • $begingroup$
    Finally the $v$ appears, but with no explanation at all?
    $endgroup$
    – NickD
    6 hours ago










  • $begingroup$
    @NickD I have left the calculation of the jacobian to the OP.
    $endgroup$
    – StubbornAtom
    6 hours ago










  • $begingroup$
    That's fine, but you should at least mention it, if only for the sake of future readers.
    $endgroup$
    – NickD
    6 hours ago










  • $begingroup$
    It is pretty clear as it is, I think.
    $endgroup$
    – StubbornAtom
    6 hours ago






  • 1




    $begingroup$
    thank you! your new parameters are better than i had chosen in question but i'm gonna choose the other answer because it directly answers to the parameters that i had mentioned in the question
    $endgroup$
    – khoshrang
    4 hours ago


















$begingroup$
Finally the $v$ appears, but with no explanation at all?
$endgroup$
– NickD
6 hours ago




$begingroup$
Finally the $v$ appears, but with no explanation at all?
$endgroup$
– NickD
6 hours ago












$begingroup$
@NickD I have left the calculation of the jacobian to the OP.
$endgroup$
– StubbornAtom
6 hours ago




$begingroup$
@NickD I have left the calculation of the jacobian to the OP.
$endgroup$
– StubbornAtom
6 hours ago












$begingroup$
That's fine, but you should at least mention it, if only for the sake of future readers.
$endgroup$
– NickD
6 hours ago




$begingroup$
That's fine, but you should at least mention it, if only for the sake of future readers.
$endgroup$
– NickD
6 hours ago












$begingroup$
It is pretty clear as it is, I think.
$endgroup$
– StubbornAtom
6 hours ago




$begingroup$
It is pretty clear as it is, I think.
$endgroup$
– StubbornAtom
6 hours ago




1




1




$begingroup$
thank you! your new parameters are better than i had chosen in question but i'm gonna choose the other answer because it directly answers to the parameters that i had mentioned in the question
$endgroup$
– khoshrang
4 hours ago






$begingroup$
thank you! your new parameters are better than i had chosen in question but i'm gonna choose the other answer because it directly answers to the parameters that i had mentioned in the question
$endgroup$
– khoshrang
4 hours ago













0












$begingroup$

Computer algebra gives (for the general case):



$$e left(x^2 text{Ei}left(-frac{2 x}{x+y}right)-frac{y^2 text{Ei}left(frac{2
y}{x+y}right)}{e^2}right)+frac{1}{2} e^{1-frac{2 x}{x+y}} (x+y)^2$$



over your specified region:



$$e-frac{1}{e}$$






share|cite|improve this answer









$endgroup$













  • $begingroup$
    yes the final answer is right due to my answer sheet but i should solve this by hand not computer any idea of how should i solve this on paper?
    $endgroup$
    – khoshrang
    7 hours ago












  • $begingroup$
    (-1) because this is not an answer to the question. See the other excellent responses!
    $endgroup$
    – Jimmy Sabater
    6 hours ago










  • $begingroup$
    @JimmySabater: I find it absolutely astounding that the "solution" (+7 and accept) lists the trivial point of how to draw a triangle, while the full correct solution leads you to downvote.
    $endgroup$
    – David G. Stork
    4 hours ago










  • $begingroup$
    The accepted solution takes the time to actually reply the steps to the asker. Your answer is not one bit helpful to be honest and when asker asked you a question you didnt even reply... that si the reason the accepted answer got many upvotes and the other answer too which also boh actually answer tbe qestion
    $endgroup$
    – Neymar
    4 hours ago
















0












$begingroup$

Computer algebra gives (for the general case):



$$e left(x^2 text{Ei}left(-frac{2 x}{x+y}right)-frac{y^2 text{Ei}left(frac{2
y}{x+y}right)}{e^2}right)+frac{1}{2} e^{1-frac{2 x}{x+y}} (x+y)^2$$



over your specified region:



$$e-frac{1}{e}$$






share|cite|improve this answer









$endgroup$













  • $begingroup$
    yes the final answer is right due to my answer sheet but i should solve this by hand not computer any idea of how should i solve this on paper?
    $endgroup$
    – khoshrang
    7 hours ago












  • $begingroup$
    (-1) because this is not an answer to the question. See the other excellent responses!
    $endgroup$
    – Jimmy Sabater
    6 hours ago










  • $begingroup$
    @JimmySabater: I find it absolutely astounding that the "solution" (+7 and accept) lists the trivial point of how to draw a triangle, while the full correct solution leads you to downvote.
    $endgroup$
    – David G. Stork
    4 hours ago










  • $begingroup$
    The accepted solution takes the time to actually reply the steps to the asker. Your answer is not one bit helpful to be honest and when asker asked you a question you didnt even reply... that si the reason the accepted answer got many upvotes and the other answer too which also boh actually answer tbe qestion
    $endgroup$
    – Neymar
    4 hours ago














0












0








0





$begingroup$

Computer algebra gives (for the general case):



$$e left(x^2 text{Ei}left(-frac{2 x}{x+y}right)-frac{y^2 text{Ei}left(frac{2
y}{x+y}right)}{e^2}right)+frac{1}{2} e^{1-frac{2 x}{x+y}} (x+y)^2$$



over your specified region:



$$e-frac{1}{e}$$






share|cite|improve this answer









$endgroup$



Computer algebra gives (for the general case):



$$e left(x^2 text{Ei}left(-frac{2 x}{x+y}right)-frac{y^2 text{Ei}left(frac{2
y}{x+y}right)}{e^2}right)+frac{1}{2} e^{1-frac{2 x}{x+y}} (x+y)^2$$



over your specified region:



$$e-frac{1}{e}$$







share|cite|improve this answer












share|cite|improve this answer



share|cite|improve this answer










answered 7 hours ago









David G. StorkDavid G. Stork

11.1k41432




11.1k41432












  • $begingroup$
    yes the final answer is right due to my answer sheet but i should solve this by hand not computer any idea of how should i solve this on paper?
    $endgroup$
    – khoshrang
    7 hours ago












  • $begingroup$
    (-1) because this is not an answer to the question. See the other excellent responses!
    $endgroup$
    – Jimmy Sabater
    6 hours ago










  • $begingroup$
    @JimmySabater: I find it absolutely astounding that the "solution" (+7 and accept) lists the trivial point of how to draw a triangle, while the full correct solution leads you to downvote.
    $endgroup$
    – David G. Stork
    4 hours ago










  • $begingroup$
    The accepted solution takes the time to actually reply the steps to the asker. Your answer is not one bit helpful to be honest and when asker asked you a question you didnt even reply... that si the reason the accepted answer got many upvotes and the other answer too which also boh actually answer tbe qestion
    $endgroup$
    – Neymar
    4 hours ago


















  • $begingroup$
    yes the final answer is right due to my answer sheet but i should solve this by hand not computer any idea of how should i solve this on paper?
    $endgroup$
    – khoshrang
    7 hours ago












  • $begingroup$
    (-1) because this is not an answer to the question. See the other excellent responses!
    $endgroup$
    – Jimmy Sabater
    6 hours ago










  • $begingroup$
    @JimmySabater: I find it absolutely astounding that the "solution" (+7 and accept) lists the trivial point of how to draw a triangle, while the full correct solution leads you to downvote.
    $endgroup$
    – David G. Stork
    4 hours ago










  • $begingroup$
    The accepted solution takes the time to actually reply the steps to the asker. Your answer is not one bit helpful to be honest and when asker asked you a question you didnt even reply... that si the reason the accepted answer got many upvotes and the other answer too which also boh actually answer tbe qestion
    $endgroup$
    – Neymar
    4 hours ago
















$begingroup$
yes the final answer is right due to my answer sheet but i should solve this by hand not computer any idea of how should i solve this on paper?
$endgroup$
– khoshrang
7 hours ago






$begingroup$
yes the final answer is right due to my answer sheet but i should solve this by hand not computer any idea of how should i solve this on paper?
$endgroup$
– khoshrang
7 hours ago














$begingroup$
(-1) because this is not an answer to the question. See the other excellent responses!
$endgroup$
– Jimmy Sabater
6 hours ago




$begingroup$
(-1) because this is not an answer to the question. See the other excellent responses!
$endgroup$
– Jimmy Sabater
6 hours ago












$begingroup$
@JimmySabater: I find it absolutely astounding that the "solution" (+7 and accept) lists the trivial point of how to draw a triangle, while the full correct solution leads you to downvote.
$endgroup$
– David G. Stork
4 hours ago




$begingroup$
@JimmySabater: I find it absolutely astounding that the "solution" (+7 and accept) lists the trivial point of how to draw a triangle, while the full correct solution leads you to downvote.
$endgroup$
– David G. Stork
4 hours ago












$begingroup$
The accepted solution takes the time to actually reply the steps to the asker. Your answer is not one bit helpful to be honest and when asker asked you a question you didnt even reply... that si the reason the accepted answer got many upvotes and the other answer too which also boh actually answer tbe qestion
$endgroup$
– Neymar
4 hours ago




$begingroup$
The accepted solution takes the time to actually reply the steps to the asker. Your answer is not one bit helpful to be honest and when asker asked you a question you didnt even reply... that si the reason the accepted answer got many upvotes and the other answer too which also boh actually answer tbe qestion
$endgroup$
– Neymar
4 hours ago










khoshrang is a new contributor. Be nice, and check out our Code of Conduct.










draft saved

draft discarded


















khoshrang is a new contributor. Be nice, and check out our Code of Conduct.













khoshrang is a new contributor. Be nice, and check out our Code of Conduct.












khoshrang is a new contributor. Be nice, and check out our Code of Conduct.
















Thanks for contributing an answer to Mathematics Stack Exchange!


  • Please be sure to answer the question. Provide details and share your research!

But avoid



  • Asking for help, clarification, or responding to other answers.

  • Making statements based on opinion; back them up with references or personal experience.


Use MathJax to format equations. MathJax reference.


To learn more, see our tips on writing great answers.




draft saved


draft discarded














StackExchange.ready(
function () {
StackExchange.openid.initPostLogin('.new-post-login', 'https%3a%2f%2fmath.stackexchange.com%2fquestions%2f3141532%2fhow-should-i-solve-this-integral-with-changing-parameters%23new-answer', 'question_page');
}
);

Post as a guest















Required, but never shown





















































Required, but never shown














Required, but never shown












Required, but never shown







Required, but never shown

































Required, but never shown














Required, but never shown












Required, but never shown







Required, but never shown







Popular posts from this blog

How did Captain America manage to do this?

迪纳利

南乌拉尔铁路局